Helicidad de antipartículas

Estoy realmente confundido por la helicidad y la manejabilidad de las antipartículas.

Considere el caso de las partículas, la solución de onda plana es ψ ( X ) = tu ( pag ) mi i pag X , dónde

tu s ( pag ) = ( pag σ ξ s pag σ ¯ ξ s ) .
Suponiendo que la partícula es ultrarrelativista y se mueve a lo largo de la + z ^ dirección, si la partícula gira hacia arriba, entonces:

tu ( pag ) = 2 mi ( 0 0 1 0 ) , h = 1 Diestro , tu ( pag ) = 2 mi ( 0 1 0 0 ) , h = 1 Zurdo ,
todo es bastante simple.

El caso de las antipartículas, ψ ( X ) = v ( pag ) mi i pag X , dónde

v s ( pag ) = ( pag σ η s pag σ ¯ η s )
con η = ( 0 1 ) y η = ( 1 0 ) . De nuevo con los supuestos de que la partícula es ultra-relativista y se mueve a lo largo de la + z ^ dirección:

v ( pag ) = 2 mi ( 0 1 0 0 ) , h = 1 ?-entregó , v ( pag ) = 2 mi ( 0 0 1 0 ) , h = 1 ?-entregó

Creo que el estado de giro hacia arriba debe ser para zurdos y el estado de giro hacia abajo debe ser para diestros, pero el giro del estado de giro parece ser paralelo al impulso, es decir, para diestros. ¿Cual es correcta?

Respuestas (4)

Si el espinor solo es distinto de cero en los dos componentes superiores, ha dejado la quiralidad . Si el espín es opuesto a su momento, tiene helicidad izquierda .

Como ejemplo, solo los campos de neutrinos con quiralidad izquierda participan en la interacción débil. Pero como acabas de ver, la quiralidad izquierda significa helicidad derecha para las antipartículas. Es por eso que a veces escuchará a la gente decir que los neutrinos son zurdos y los antineutrinos son diestros. Están hablando de helicidad en ese caso.

Entonces, para ser claros, es ambiguo qué poner en lugar de los signos de interrogación en su pregunta. Si hablamos de quiralidad, v queda y v es correcto. Si hablamos de helicidad, v es correcto y v es izquierda.

Entonces, ¿significa esto que tanto el neutrino como el antineutrino tienen helicidad izquierda ( h = 1 )?
No, ambos han dejado la quiralidad. Los neutrinos tienen helicidad izquierda y los antineutrinos, helicidad derecha. Estoy de acuerdo en que la terminología es confusa cuando la gente dice derecha e izquierda y no especifica qué concepto.
Una cosa a tener en cuenta es que si cuantificamos el campo de Weyl quiral izquierdo, se convierte en una combinación lineal de un término que aniquila una partícula levógira y otro término que crea una antipartícula levógira. Así que decir que tanto la partícula como la antipartícula son quirales por la izquierda no me suena bien.
De hecho, nada nos impide construir un campo de Weyl cuántico de quiral derecha a partir de operadores de aniquilación de antipartículas y operadores de creación de partículas, aunque será bastante complicado representar la interacción débil usando este objeto.
@higgsss, en mi respuesta dije que solo el campo quiral izquierdo participa en la interacción débil. Cual es verdad. Sí que implica una convención sobre cuál es la partícula y la antipartícula. Pero a veces, cuando respondo una pregunta, creo que ayuda mantener la respuesta simple, aunque entiendo si no te gusta aplicar la palabra quiral a estados en lugar de campos o espinores.
Entiendo. Estaba un poco preocupado por su primer comentario, que dice que tanto el neutrino como el antineutrino han dejado la quiralidad.

La interpretación de la teoría del agujero de Dirac es útil aquí. La solución v ( pag ) mi i pag X = v ( pag ) mi i k z mi i ω t con ω = metro 2 + k 2 es un estado de momento de partícula de energía negativa k z ^ , energía ω , y S z = 1 / 2 . Uno debería aniquilar este estado del vacío para obtener el correspondiente estado antipartícula de energía positiva .

El resultado de aniquilar el impulso. k z ^ , energía ω , y S z = 1 / 2 del vacío es un estado de impulso k z ^ , energía ω , y S z = 1 / 2 . Entonces, el estado de antipartículas en cuestión es diestro.

Para mayor claridad, uno podría evitar usar las palabras "derecha" e "izquierda" por completo, y hablar de quiralidad positiva y negativa; helicidad positiva y negativa.

La helicidad es +ve si σ pag > 0 y negativo si σ pag < 0 .

Para un espinor de Weyl, la quiralidad tiene un signo si los elementos de la matriz del espinor s | σ a | s = tu a dar las componentes contravariantes (en oposición a covariantes) de un cuadrivector, y la quiralidad de | s tiene el otro signo si s | σ a | s = tu a , es decir, componentes covariantes. Pero es una cuestión de convención qué caso se dice que es de quiralidad positiva o negativa, y es una cuestión de convención si se dice que la quiralidad positiva es "derecha" o "izquierda". Creo que la convención es que "correcto" se asocia con "positivo", que a su vez se asocia con "contravariante".

En esta convención, un espinor de Weyl de quiralidad positiva, cuando se escribe como un vector complejo de dos componentes, se transforma como

s Λ s
bajo un cambio de marco de referencia, donde la transformación de Lorentz
Λ = Exp ( i σ θ / 2 σ ρ / 2 )
en el cual σ es el trivector de las matrices de Pauli, θ es un vector de tres que define un eje y la cantidad de rotación, y ρ es el tres vector de rapidez.

(Incluí esta información sobre Λ para ser precisos sobre la terminología en esta respuesta).

Volviendo ahora a los espinores de Dirac, creo que una convención común pero no universal en la representación quiral es poner el espinor de Weyl de quiralidad negativa en la parte superior y el espinor de Weyl de quiralidad positiva en la parte inferior. Así uno ve cosas como:

( ϕ L x R ) .
Esta es la convención implícita en la respuesta proporcionada aquí por octonion. Sin embargo, creo que puede ser útil notar que alguna literatura puede adoptar la otra convención posible y poner x R en la cima.

Bajo la convención anterior, las matrices de Dirac leen

γ 0 = ( 0 I I 0 ) , γ i = ( 0 σ i σ i 0 )
pero también los he visto escritos en la versión transpuesta de esto, lo que implica que el espinor de Dirac se está escribiendo al revés.

Creo que la única forma de acertar todos estos signos en el propio trabajo es darse cuenta de que no es fácil y luego rastrear minuciosamente sus propias expresiones hasta el punto en que puede hacer una transformación de Lorentz y ver lo que cree que sucedió con el impulso de su partícula.

Para una partícula, la helicidad y la quiralidad son idénticas, mientras que para una antipartícula, la helicidad y la quiralidad son opuestas en el caso sin masa. Entonces, para partículas sin masa que tienen quiralidad zurda, también tendrán helicidad zurda. Para la antipartícula sin masa que tiene quiralidad levógira, tendrá helicidad levógira. Este hecho se basa en el uso de la ecuación de Dirac para partículas/antipartículas sin masa.